Forgetting my inverse tangent, polar form of compelx number ASTC

Click For Summary
SUMMARY

The discussion focuses on finding the polar form of the complex number \(2i - 1\). The correct polar form is derived using the formula \(r(\cos \theta + i \sin \theta)\), where \(r = \sqrt{a^2 + b^2}\) and \(\theta\) is determined using the arctangent function. The initial calculation of \(\theta = \arctan(-2)\) is incorrect, as the angle should be adjusted to \(116.57^\circ\) based on the quadrant of the complex number located at \((-1, 2)\). The quadrant system (ASTC) is essential for determining the correct angle.

PREREQUISITES
  • Understanding of complex numbers and their representation in Cartesian form
  • Familiarity with polar coordinates and conversion from Cartesian to polar form
  • Knowledge of trigonometric functions, specifically sine and cosine
  • Ability to use the arctangent function for angle calculation
NEXT STEPS
  • Study the polar form of complex numbers in detail
  • Learn about the ASTC (All Students Take Calculus) rule for determining signs of trigonometric functions in different quadrants
  • Practice converting various complex numbers from Cartesian to polar form
  • Explore the implications of angle adjustments in trigonometric calculations
USEFUL FOR

Students studying complex numbers, mathematics educators, and anyone looking to strengthen their understanding of polar coordinates and trigonometry.

thomas49th
Messages
645
Reaction score
0

Homework Statement


Find the polar form of 2i − 1


Finding polar form is easy r(cosx + isinx)

call the real part a and imaginary part b

r = sqrt(a+b)

theta = arctan (-2) = - 63.43

This is the wrong angle for theta as it's 116.57 (which is 180 - 64.43), and I guess this if form that quadrant thing

S | A
___|___
|
T | C


okay not the best grid, but can anyone refresh my memory as how to get the correct value of theta?

Thanks :)
 
Physics news on Phys.org
Hi thomas49th! :smile:

(have a theta: θ and a square-root: √ :wink:)

(and you meant √(a2 + b2))

Yes, your table shows what's positive (sin, tan, cos, or all) …

(it works because cos*tan = sin, so an even number must be negative, so either 1 or all 3 must be positive :wink:)

so arctan(-2) can be either top left or bottom right, you can't tell which by looking at it

you have to go back to the original 2i - 1, which is at (-1,2), so it's obviously top left! :smile:
 
Question: A clock's minute hand has length 4 and its hour hand has length 3. What is the distance between the tips at the moment when it is increasing most rapidly?(Putnam Exam Question) Answer: Making assumption that both the hands moves at constant angular velocities, the answer is ## \sqrt{7} .## But don't you think this assumption is somewhat doubtful and wrong?

Similar threads

  • · Replies 5 ·
Replies
5
Views
6K
  • · Replies 8 ·
Replies
8
Views
4K
  • · Replies 14 ·
Replies
14
Views
2K
  • · Replies 9 ·
Replies
9
Views
4K
  • · Replies 8 ·
Replies
8
Views
3K
Replies
2
Views
3K
  • · Replies 3 ·
Replies
3
Views
3K
Replies
2
Views
3K
  • · Replies 4 ·
Replies
4
Views
3K
  • · Replies 175 ·
6
Replies
175
Views
26K